Email
Chat with tutors
logo

Ask Questions, Get Answers

X
 
Questions  >>  CBSE XII  >>  Math  >>  Model Papers
Answer
Comment
Share
Q)

For what value of k , the matrix \( \begin{bmatrix} k & \quad 2 \\ 3 & \quad 4 \end{bmatrix} \) has no inverse.

Please log in or register to answer this question.

Home Ask Tuition Questions
Your payment for is successful.
Continue
...